Difference between revisions of "1992 AHSME Problems/Problem 7"

(Solution)
m (Small correction)
 
Line 27: Line 27:
 
<math>\frac{w}{y} = \frac{16}{3}</math>
 
<math>\frac{w}{y} = \frac{16}{3}</math>
  
<math>w</math> to <math>x</math> is <math>16:3</math>
+
<math>w</math> to <math>y</math> is <math>16:3</math>
  
 
== See also ==
 
== See also ==

Latest revision as of 02:24, 20 February 2018

Problem

The ratio of $w$ to $x$ is $4:3$, of $y$ to $z$ is $3:2$ and of $z$ to $x$ is $1:6$. What is the ratio of $w$ to $y$?

$\text{(A) } 1:3\quad \text{(B) } 16:3\quad \text{(C) } 20:3\quad \text{(D) } 27:4\quad \text{(E) } 12:1$

Solution

$\fbox{B}$

Converting from ratios to fractions

$\frac{w}{x}=\frac{4}{3}$, $\frac{y}{z}=\frac{3}{2}$, $\frac{z}{x}=\frac{1}{6}$


$\frac{y}{z} \cdot \frac{z}{x} = \frac{3}{2} \cdot \frac{1}{6}$

$\frac{y}{x} = \frac{1}{4}$

$\frac{(\frac{w}{x})}{(\frac{y}{x})}=\frac{\frac{4}{3}}{\frac{1}{4}}$

$\frac{w}{y} = \frac{16}{3}$

$w$ to $y$ is $16:3$

See also

1992 AHSME (ProblemsAnswer KeyResources)
Preceded by
Problem 6
Followed by
Problem 8
1 2 3 4 5 6 7 8 9 10 11 12 13 14 15 16 17 18 19 20 21 22 23 24 25 26 27 28 29 30
All AHSME Problems and Solutions

The problems on this page are copyrighted by the Mathematical Association of America's American Mathematics Competitions. AMC logo.png